« first day (43 days earlier)      last day (531 days later) » 
00:00 - 16:0016:00 - 00:00

4:00 PM
0
Q: Find the left cosets of subroups of S3

Lauren BathersSo I am struggling to understand the definition of a coset. If I have the following symmetric group $S3=${1$, \sigma, \sigma\tau, \sigma\tau^2, \tau, \tau^2$} where $\sigma$=($\begin{array}{ccc}1 & 2 & 3 & \\ 1 & 3 & 2 \end{array}$) and $\tau$=($\begin{array}{ccc}1 & 2 & 3 & \\ 2 & 3 & 1 \end{arr...

Title ends with a digit. Find the left cosets of subroups of S3
 
0
Q: Two votes on this question to close as “not programming related” – how best to respond, as the asker?

fish2000I asked this clearly programming-related question: Smart pointer analog of `std::shared_ptr` with API to bind callbacks to refcount-modifying events e.g. release/retain … is this a thing? … which has been met with two close-votes, both as “not programming related”. I really don’t quite know how ...

0
Q: Should I close my question if I've upgraded the problem scenario away?

JoshDMI asked a question where I have outlined a very specific communication scenario between two systems using two specific versions of software (the question; no arbitrary voting, please). One of the systems has been upgraded to a much higher version, and it solved the problem; I can no longer test ...

 
0
Q: Flat closed universe

Jorge MaxWe usually read of our universe as a flat infinite universe (in accordance with WMAP and Planck data). Yet, it is possible to have a flat closed universe with a finite volume. One clear example of this is a 3D Torus. What are the basic parameters of this body and what is its volume as a function ...

Short title. Flat closed universe
0
Q: How to use Farkas' lemma?

AerrowSHow can I prove, that the set $$P = \{(x, y) \in \mathbb{R}^{n+m} : Ax + By \geq c, \: x \geq 0^n, \: y\geq 0^m \}, $$ where $B \in \mathbb{R}^{m \times m} \;$ is positive semidefinite matrix, $A \in \mathbb{R}^{m \times n} \;$ and $c \in \mathbb{R}^{m}$, is compact only when $P = \emptyset$ ? ...

0
Q: Problem with $P(\liminf(A_n\cup B_n))$

RErWe know that $P(\liminf_n A_n)=0.3$ and $P(\limsup_n B_n)=0$. Find $P(\liminf(A_n\cup B_n))$. My solution: We know that $\liminf_n(A_n\cup B_n)\supset \liminf_nA_n \cup \liminf_n B_n \supset \liminf_n A$. Therefore we get $$P(\liminf_nA_n)=0.3\leqslant \liminf(A_n\cup B_n).$$ How should I show t...

 
-3
Q: The meaning of "open-ended"

D_BesterHow is this open-ended? How do I restrict the use of international holidays in a date entry cell? I've looked at lots of open-ended questions. In my judgement this is not a broad question at all. Basically they are asking how to use data validation in Excel to NOT allow a list. How is this open...

 
0
Q: Integral $\int^l_{-l}{r/({\sqrt{L^2+r^2}})^3}dL$

zanewe try to solve a magnetics problem and it end up with $\int^l_{-l}{r/({\sqrt{L^2+r^2}})^3}dL$ I don't know how to get its integral .

0
Q: Example for Integral closure

SelvaLet $R=k[x^4,x^3y,xy^3,y^4]$ be a polynomial ring. We can see $R$ is not integrally closed (Since $x^2y^2 \in Q(R)$ is integral over $R$ but $x^2y^2 \notin R$ ). Therefore $k[x^4,x^3y,x^2y^2,xy^3,y^4]\subseteq\bar{R}$ (where $\bar{R}$ is integral closure). How to show that otherside?

0
Q: Recurrence of $\int^t_0W_s ds$, where $W$ is a Brownian Motion

Lost1Is there any easy way of showing this integral is recurrent? i.e. it visits every point infinitely many times?

0
Q: Is the norm map in a class formation always of finite kernel?

jdhDoes the norm map in a class formation always have a finite kernel?

Questions tend to get more attention when they have a tag for a broad area of mathematics relevant to the question. Some of these tags might fit. (from a bot)Normal Human 21 secs ago
 
4:22 PM
-2
Q: Allow anyone to downvote un-relevant tags

jimmy obonyoId like to recommend users below 2000 points to be able to down-vote new un-relevant tags , i personally have come across a tag which is completely un-relevant as it refers to an amount in ksh ie ksh93 the tag link in question is http://stackoverflow.com/questions/tagged/ksh93 .

 
0
Q: please help to solve this integral

solafidefrom stein exercise 6-5 Let A be a dxd positive definite symmetric matrix with real coefficient show that integral [e^(-pi*(x,A(x))] dx = det(A)^(-1/2) (integral is on R^d , so x is) hint is applying spectral thm that A=RDR^-1, where R is a rotation and D is diagonal matrix maybe it is not ...

Words such as please, help are uninformative in titles. Please edit the title so that it better describes the specifics of your question. Do not hesitate to make it longer or include a formula if needed. This site uses MathJax formatting of formulas. More tips here. (from a bot)Normal Human 21 secs ago
0
Q: Is the distribution of a random variable always a probability measure?

JackIs the distribution of a random variable always a probability measure? Let $\mathbb{P}X^{-1}$ denote the distribution of the random variable $X$.

 
0
Q: Do bounties have to reward answers only?

AlexIf you have a bounty on a question, and somebody posts a comment on that question, which is a link to an answer on another question, and that is the solution you're looking for, how do you reward the person who posted the comment with the link? Considering that just posting the link as an answer ...

 
0
Q: Question to find the value of P in following question

AyushakjLet a, b and c be such that a + b + c = 0 and P = $a^2/(2a^2 + bc)$ + $b^2/(2b^2 + ca$) + $c^2(2c^2 + ab)$.How could I find Integral value of P.

Words such as question, question do not add information to titles. Please edit the title so that it better describes the specifics of your question. Do not hesitate to make it longer or include a formula if needed. More tips here. (from a bot)Normal Human 21 secs ago
0
Q: Acceptance by final state PDA and acceptance by empty stack .

user123Let P be a non-deterministic push-down automaton (NPDA) with exactly one state, q, and exactly one symbol, Z, in its stack alphabet. State q is both the starting as well as the accepting state of the PDA. The stack is initialized with one Z before the start of the operation of the PDA. Let the in...

Welcome to Math.SE, user123. This site uses MathJax formatting of formulas. More tips here. (autocomment)Normal Human 21 secs ago
0
Q: Identity theorem for power series

Raheem Najib I spent a lot of time but I can't understand why $B$ is open? Can anyone show it rigorously, please?

Short question. Question contains please. Identity theorem for power series
0
Q: question on geometry,circle containing circle

Ayush JaiswalThe circle ω touches the circle Ω internally at P. The centre O of Ω is outside ω. Let XY be a diameter of Ω which is also tangent to ω. Assume PY > PX. Let PY intersect ω at Z. If Y Z = 2PZ, what is the magnitude of angle PYX in degrees?

Words such as question are uninformative in titles. Please edit the title so that it better describes the specifics of your question. Do not hesitate to make it longer or include a formula if needed. More tips here. (autocomment)Normal Human 21 secs ago
 
4:43 PM
0
Q: How to gather support for tag synonyms?

Bhargav RaoI proposed a tag synonym a few days ago. Out of curiosity I looked at the tag synonym page and found out that no one as voted. I later found out that there are suggestions pending approval from mar 31 '12 which is more than Three and a half years old. Hence my question is, Is there a way t...

-9
Q: The good in bad questions

Mike159I guess this essentially boils down to 'quality questions vs helping noobs' or 'does SO suffer from elitism'. (And turned out not to really be a question) The SO 'mission statement' (if you would call it that) said "Better programming is our goal". But where does better programming come from, go...

0
Q: how to correct an erroneous edit

roy smith"Black adder" edited my answer to a question asking for a proof of noether normalization. the edit was nicely done, except in the next to last sentence a change was made from "≤ d" to "less than d", which introduces an error. I do not know how to either fix it or message the editor of the quest...

 
0
Q: Calculating the order of n modulo m from a congruence equation

user286417We are given an integer $n$ s.t. $n^2+n+1 \equiv 0 \mod m$ Where $m>3$ is a prime. Find$|n|_{m}$ and show that $m \equiv 1 \mod 3$

0
Q: Show that K[X] is UFD

NesaHow to show that (if K is a field) then K [x] is a unique factorization domain? And why does it follow from Gauss's lemma that (since $Q$ is a field) Z[x] is also a UFD.

Short title. Short question. [Show that K[X] is UFD](math.stackexchange.com/q/1577059)
0
Q: Asymptotic notation with different class of O(f)

geoFor f ∶ N → R+ other class of f, O′(f) ∶= {g ∶ N0 → R+ ∥ ∃c > 0, ∀n ∈ N0 ∶ g(n) ≤ c ⋅ f(n) + c} Proof that: When limn n→∞ f(n) > 0, then g ∈ O(f) ⇔ g ∈ O′(f).

Welcome to Math.SE, geo. Consider replacing (analysis) with a more specific tag for the relevant branch of analysis. (from a bot)Normal Human 21 secs ago
0
Q: How are subtraction operations better conditioned than addition?

mohakbhI was reading about error analysis in numerical methods, particularly about calculating the sum of arrays. I understand how naively summing all elements can lead to accumulation of error especially when the number of elements is very large and all elements are of small order. In the alternative ...

Questions tend to get more attention when they have a tag for a broad area of mathematics relevant to the question. Some of these tags might fit. (autocomment)Normal Human 20 secs ago
0
Q: the $\partial\bar{\partial}$-lemma dilemma

PhilosophicalPhysicsIn the question here Simplifying the Kahler form, user290605 asked a question about how is that when we take the differential of Kahler form:$$\mathcal{K}=\frac{\sqrt{-1}}{2\pi}g_{i\bar{j}}dz^i\wedge d\bar{z}^{\bar{j}},$$ we get$$\partial_ig_{j\bar{k}}=\partial_jg_{i\bar{k}} \hspace{1cm} \text{an...

 
5:12 PM
1
Q: A quick new nav update

Ebenezer SklivvzeTL;DR: please help us test this version of new nav before we begin rolling it out to more unsuspecting victims nice users for A/B testing. Since the last RC we have introduced the following requested changes: tabs are saved on the server, and will be the same across all browsers the page is ...

 
0
Q: suggest me some books on Number theory

AyushakjWhich can be the best book for elementary Number Theory of High School students or, to get started with Number Theory which book should I read. Please suggest.

Short question. Question contains please. suggest me some books on Number theory
0
Q: neumaan's formula

Master Mathematicianneumann's formula : $$Q_{n}(z)= \frac{1}{2}\int \frac{P_{n}(\mu )}{\mu -z}d\mu $$ prove that: $$P_{n+1}(z)*Q_{n}(z)=\frac{1}{2}\int \frac{P_{n+1}(\mu )P_{n}(\mu )}{z-\mu }d\mu +\frac{1}{n+1}$$ solution : $$z^{n+1}Q_{n}(z)-\frac{1}{2}\int_{-1}^{1}\frac{\mu ^{n+1}P_{n}(\mu )}{z-\mu }d\mu =\frac{...

Short title. neumaan's formula
0
Q: Implicit Solution to a differential equation

Greg.PaulI'm looking at the ODE: $\frac{dY}{dX} - \frac{ X^2 + 2 Y^2 - 1 }{ ( Y - 2 X )X } = 0$ I'm looking for an $implicit$ solution to the above. Meaning, I want to find a relation $F(X,Y)=0$, where $\frac{\partial}{\partial x} (F(x,y)) = \frac{dY}{dX} - \frac{ X^2 + 2 Y^2 - 1 }{ ( Y - 2 X )X }$. $\...

Tagged differential-equations but mentions "partial". Implicit Solution to a differential equation
0
Q: Lower Reimann integral

Al jabraI am reading Elements of Integration by Bartle and I came across this. "If f is a bounded function defined on an interval [a,b] and if f is not too discontinuous .......In particular the lower Reimann integral of f may be defined to be the supremum of the integrals of all step functions $\gamma$...

 
0
Q: Achievements doesn't show rep change on closed question

Tʀᴇᴠɪ AᴡᴀᴛᴇʀThis morning at 22:45 I received an up-vote on my answer to this closed question. For some reason it does not show up in the achievements tab, is this a bug or am I missing something?

 
0
Q: One-to-one function

aazz20How can I determine if the function is one-to-one .. I know that any odd function is 1-1 and any even function is NOT 1-1 but what about functions thats are nither like X^3+5 and X^3+x^2+3 How can I determine whether it is a one-to-one ?

This site uses MathJax formatting of formulas. More tips here. (autocomment)Normal Human 21 secs ago
0
Q: A problem on subspaces

TurboSupposing we have a space $S$ in $2n$-dimensions. You have two sets of $n$ number of indepenent constraints $C_1$ and $C_2$ but that are related to each other (that is if you arrange them as a $2n\times 2n$ matrix rank is $n$). Consider space corresponding to $C_1$ be $S_1$ and consider space c...

0
Q: Can't get to solve this -- word problems

MohammedPrice of lemon juice bottle is 4 , price of orange juice bottle is 6 A buyer bought 20 bottles and the total cost is 96 How many lemon bottles and orange bottle did the buyer get ? I know the answer but i don't know the steps to get to it ... Please answer the question with steps Thanks

Title contains problem. Question contains please. Can't get to solve this -- word problems
Consider adding a tag for a broader subject area to which the question belongs. Some of these tags might fit. (from a bot)Normal Human 20 secs ago
0
Q: If product of matrices $A$ and $B$ is defined, then $rank(AB)=rank(BA)$

aprilduckIf product of matrices $A$ and $B$ is defined, then $rank(AB)=rank(BA)$. Is this always true, or just in some special cases?

 
5:32 PM
0
Q: Why sometimes the number of total-votes is different between various communities?

ShafizadehI think the algorithm of calculating the number of votes for each question/answer is different between different communities. Here is one of my question on the StackExchange: As you see, StackExchange shows the number of total_votes of that question is 6. Here is the same question in the sta...

 
0
Q: Does the following set is open? closed? or none?

CrazyStatisticianLet: $$ B:= \{ (x,y)|-1\leq y\leq 1 \} $$ Is it correct to say that $B$ is closed, because: $$ B^C := \{ (x,y) | |y| >1 \} $$ is open? Thanks !

0
Q: Tricky convergence questions $\sum_{n=1}^\infty {\frac{n+2}{n^3-2n^2+1}}$

babylonDo the following converge: $\sum_{n=1}^\infty {\frac{n+2}{n^3-2n^2+1}}$ For this one I think the answer is no I just can't prove it. I split it up into partial fractions and got: $\frac{3n+1}{n^2-n-1}-\frac{3}{n-1}$ but after that I'm stumped :( The second part: $\sum_{n=1}^\infty {\frac{1}{n...

0
Q: Proof about the general form of invariant subspaces of nilpotent operators

Soba noodlesSome while ago I encountered a theorem which goes like this: Let $V$ be a $n$-dimensional vector space, and $N:V\to V$ a (linear) nilpotent operator of index $n$. Suppose we're given a cyclic base $E=\{e,Ne,...,N^{n-1}e\}$, where $e\in V$. Then all nontrivial invariant subspaces of $N$ are given...

0
Q: Mathematics doubts homework

SamagraI posted my question on math.stackexchage . But many replied that you cannot post a homework . So can please tell me other sites on which I can post my maths homework for free and the reply is fast .

Short title. Title contains homework. Question contains please. Mathematics doubts homework
Please don't use (self-learning) tag just because you were self-studying. This tag is only for questions about the process of self-studying. (autocomment)Normal Human 20 secs ago
0
Q: How can i show why this linear recurrence relation is satisfied? And why it works?

MichaelWe have a sequence of n numbered parking spaces which are arranged in a line. Type A vehicles require one parking space and Type B vehicles require two parking spaces. Let H(n) denote the number of ways in which the n spaces can be filled. a) If $ n \ge 3 $, explain why if the first car is Type ...

0
Q: Simultaneous linear congruence

user298943I'm following a sample solution from my lecturer and part of it is to solve $x \equiv 9 (mod 12) $ and $x \equiv 3 (mod7) $ simultaneously. He writes it as $9 + 12k \equiv 3 (mod 7)$ which I understand He then writes: This can be rewritten as $5k \equiv 1 (mod 7)$ I can see how to solve it fr...

0
Q: How to find the limit $\lim_{x\to0}\cfrac{x-x\cos x}{tan^3x}$ without using the L'hopital rule or the taylor series?

Daniel AndradeI found that exercise. However I can not find the answer. I know I can do it with L'hopital. However is there a way to solve it without using L'hopital nor Taylor series? If so show me please. I will appreciate it a lot. If there's no way, which solution seems easier? L'hopital?Taylor?

0
Q: How to prove $G$ is cyclic?

tempuraGiven that $G$ is a finite abelian group, and for every prime $p$ that divides the order of $G$, there is a unique subgroup of order $p$. How can I prove that $G$ is cyclic?

Short title. Short question. How to prove $G$ is cyclic?
0
Q: Finding the Fourier Series of a function u(x,t)=x

user286417It is a pretty basic function but I am still having problems. Expand the following function $U(x,t) = x$ into a Fourier Series on $(-1,1)$

 
5:51 PM
0
Q: Where's my Sufganiyot hat?

RobHThe description for the Sufganiyot hat says "post or vote on Dec 14." I voted on something yesterday (I believe it was in SO meta), but I didn't get the hat. What's up with that?

0
Q: Circular motion rotation

SamagraA spherical tank of 1.2 m radius is half filled with oil of relative density 0.8. If the tank is given a horizontal acceleration of 10 m/s2. Calculate the inclination of the oil surface to horizontal and maximum pressure on the tank.

 
0
Q: Proof Check in Algebra

GeorgeI was resolving a question about partition, quotient set and equivalence relation, but I don't sure if my proof is correct. Anyone can help me? I'm grateful right now for all help. Follows the question and my attempt, respectively. Question: Let be $A$ an set non-empty and $P(A)$ power set of $...

Short title. Tagged proof-verification. Proof Check in Algebra
0
Q: Frattini subgroup of a finite elementary abelian $p$-group is trivial

BungoI would like to improve my proof of the following result: If $H$ is a finite, elementary abelian $p$-group, then $\Phi(H) = 1$. Here, $\Phi(H)$ is the Frattini subgroup, defined as the intersection of all maximal subgroups of $H$. An elementary abelian $p$-group is an abelian group with the...

0
Q: How to solve this dynamic logic problem? I think only (a) and (d) holds for all ui, is that so?

Chen LuLet p and q be atomic propositions and a and b be atomic actions. S={u1,u2,u3,u4} R(a)={(u1,u1),(u2,u1),(u4,u1)} R(b)={(u2,u3),(u2,u4),(u3,u3),(u4,u2),(u4,u3)} π(p)={u1,u2} π(q)={u1,u4} Let M be the dynamic structure specified above. There are 4 states given by the elements of S. a and b a...

 
6:07 PM
0
Q: Question in Lattice Reduction Using LLL Algorithm

Ahmed Suppose we have a Lattice $L$ in $R^n$ generated by a basis $L(b_1,...,b_d)$, where $b_i$ is a column vector with $n$ elements, assume $d<n$, could we reduce the basis $L(b_1,...,b_d)$ to new bases $L(b^*_1,...,b^*_d)$ using $LLL$ algorithm? Thanks in advance

Words such as question do not add information to titles. Please edit the title so that it better describes the specifics of your question. Do not hesitate to make it longer or include a formula if needed. Questions tend to get more attention when they have a tag for a broad area of mathematics relevant to the question. Some of these tags might fit. More tips here. (from a bot)Normal Human 21 secs ago
 
2
Q: Question list stops being expanded when switching to new tab from home page

resuemanWhen I go to the home page of either SO or Meta (i.e. http://stackoverflow.com/), it shows all the questions in the expanded view. From there, if I click on any tab (other than home) it switches to the collapsed view, instead of staying in expanded view. I think the intended behavior is that the ...

 
0
Q: Problem with $\mathbb{P}(\liminf_n(A_n\cup B_n))$

RErWe know that $\mathbb{P}(\liminf_n A_n)=0.3$ and $\mathbb{P}(\limsup_n B_n)=0$. Find $\mathbb{P}(\liminf_n(A_n\cup B_n))$. My solution: We know that $\liminf_n(A_n\cup B_n)\supset \liminf_nA_n \cup \liminf_n B_n \supset P(\liminf_n A_n)$. Therefore $$\mathbb{P}(\liminf_nA_n)=0.3\leqslant \liminf...

0
Q: Proof for $A^2=I$ if $A$ is diagonalizable and $A^m=I$ for some $m\ge 1$.

CoolKidSo if A is diagonalizable, there exists $PAP^{-1}=D$ and also $PA^mP^{-1}=I$. To prove $A^2=I$, we need $D^2=I$ but how does it work?

0
Q: Open sets Are balls?

Manolis LyviakisIs any open set a ball? So instead of open sets we can talk about balls or disks? Is that correct? Because ihave seen definitions with open sets Like the definition of a surface.So is it sufficient instead of talking about generally open sets simplify things thinking it as balls?

Short title. Open sets Are balls?
0
Q: Chromatic number for random graph

AcapelloBuilding random graph with probability to connect two vertex $p = \frac{1}{2n}$, and not connect $q = 1 - \frac{1}{2n}$. Find chromatic number a.a.s.(asymptotically almost shure), when $n$ tends to infinity. I've tried to use that $\chi(G) \geq \frac{n}{\alpha(G)}$ to get lower bound, but I fai...

Question contains please. Chromatic number for random graph
0
Q: Enrichment / over-representation of pairs, triplets, quadruples, etc. (a bit tricky and challenging statistical test)

AdamI would like to create statistical test that detects over-representation / enrichment of structures such as pairs, triplets, quadruplets, etc. in a given group of obejcts compared to another group. The tricky part is to take into account the substructures. For example, let's say I'm interested ...

0
Q: Basic question on the convergence domain of a Laurent Series,

user298944If the principal part converges for, say, |z|>1, and the analytic part (the positive powers in $z-z_0$) converges for |z|<2, then does the Laurent series, as a whole, converge in the annulus 1<|z|<2? So, this would be like taking the overlapping region of convergence of the principal part and th...

0
Q: Computable problem

user159870A mathematical problem is computable if there is an algorithm that decides this problem, right? Can you give an example of such a problem?

Short title. Title contains problem. Short question. Computable problem
0
Q: when an area of trapezoid ABCD reaches maximum , sum of two length of diagonal =?

user157844When an area of trapezoid ABCD reaches maximum , what is length of sum two diagonal ?

0
Q: Bianalytic endomorphisms of $\mathbb{C}-\{0,1\}$

HansI have been searching for complex analysis problems recently. In https://www.math.stonybrook.edu/~olga/mat542/hw8_542.pdf I found the problem to determine the bianalytic endomorphisms of $\mathbb{C}-\{0,1\}$. Could someone give me some hints?

 
6:44 PM
0
Q: Not an Answer flag declined while most reviewers seem to agree on Deletion

AthafoudEarlier today my 'Not an Answer' flag for this was declined as "a moderator reviewed your flag, but found no evidence to support it". Looking through the review queue, it seems that the reviewers 'agree' (as I can understand it) to delete this question. So why my flag was declined? Do I und...

 
0
Q: Residually Finite Braid Group

MonsieurGaloisIn Braid Groups of Kassel, Turaev, it mentions that $\mathcal{B}_n$ is a residually finite group. The definition that they give as a residually finite group is a group $G$ such that for each $g\in G-\{e_G\}$ ($e_G$ the identity of $G$), there exists an homomorphism $f$ to a group $H$ such that $f...

0
Q: What is Calculus?

elika kohenNote: The hope here is for a definition that does not rely on "Jargon", and efficiently conveys the nature of "Calculus". Question: What is a practical method to Define and Illustrate "Calculus" that could be provided, while either avoiding or clarifying "Mathematical Jargon" seen in common de...

Short title. What is Calculus?
0
Q: Difference bbetween order and cardinal

Merie Marissalet G be a group and H a subgrouop I want to know the difference between the order and the cardinal of a subgroup

0
Q: Interesting puzzle question

fillhoraHere is an puzzle question in this website. I hope someone will be interested in and solve. http://www.gizmodo.com.au/2015/12/can-you-solve-the-uk-intelligence-agencys-christmas-puzzle/

Words such as interesting, question do not add information to titles. Please edit the title so that it better describes the specifics of your question. Do not hesitate to make it longer or include a formula if needed. More tips here. (autocomment)Normal Human 20 secs ago
0
Q: Reimann Definite Integration

Soledolu Oluwatayoplease i really need help on this express lim(n->∞) {1/n+1 + 1/n+2 + ... +1/n+n } as reimann integral. I was able to transform it to him(n->∞)£{1/n+r} I don't know how to go about it

This site uses MathJax formatting of formulas. More tips here. (from a bot)Normal Human 20 secs ago
0
Q: A vector field with specified curl

João JúniorI need a vector field $\vec{F}:\mathbb{R}^3\to\mathbb{R}^3$ such that $$\mathrm{curl}\ \vec{F}(x,y,z) \cdot \left(\frac{-x}{\sqrt{x^2+1}},\ 0,\ \frac{1}{\sqrt{x^2+1}}\right) = 1.$$ (This equality, of course, holds if $\mathrm{curl}\ F(x,y,z) = \left(\frac{-x}{\sqrt{x^2+1}},\ 0,\ \frac{1}{\sqrt{x...

Questions tend to get more attention when they have a tag for a broad area of mathematics relevant to the question. Some of these tags might fit. (from a bot)Normal Human 20 secs ago
0
Q: Center of noetherian rings

alex alexeqIs it true that the center of a right Noetherian ring (with identity) is always Noetheria. ?

Short title. Short question. Center of noetherian rings
0
Q: Linear Independence of a Span

Juan Manuel CastroIf I have $({u,v,w})$ is a linear independent set, then for which values of $\alpha,\beta$ is {${u,\alpha u +v,\beta v +w}$} a linear independent set? I claim that for all $\alpha,\beta$ in $\mathbb{R} $ this holds. Is this true?

0
Q: $\lim 2^{\frac{1}{\sqrt{n}}} =1 $

ShlomiProve that $$ \lim_{n\to\infty} 2^{\frac{1}{\sqrt{n}}} =1 $$ I tryed to solve this using the sandwich theorem and the fact that $\lim 2^{\frac{1}{{n}}} =1 $. We did not learn continuity of functions so I am not alowed to use this. Any suggestions?

 
7:03 PM
0
Q: What query do I need to create to check which questions are related to my progress in Socratic badge?

Bartłomiej SemańczykSo far I have 59 of 100 to earn Socratic badge. Is there a way which questions helped me to earn that badge? I need to know the dates when these questions were created.

0
Q: Search Results Count Missing

War10ckThere appears to be a bug on the SO search results page. Admittedly, I don't use search that often so I may be mistaken but it appears that the number of results returned is not displayed in the right hand sidebar. If a number is never shown there and this is normal behavior, is the "gap" of w...

 
A title should not be all-MathJax; having some plain text helps with search and navigation. (autocomment)Normal Human 20 secs ago
0
Q: Proving Monotonicity Slickly

HansWhat are some slick ways to prove, with respect to $x\in[0,\infty)$, $S_1(x)$ is increasing, and given $b\in[\frac12,1]$ $S_2(x)$, is decreasing as defined below? I have proved it by taking several rounds of derivatives for various functions in the intermediate steps. That is not very elegant. \...

0
Q: double integration of (x-y) u = 0 w.r.t x, y

Saimawhat is the solution of this equation. Please help me. ∬[(x-y)u]dxdy = ? where, (x-y)u = 0

Short question. Question contains please. double integration of (x-y) u = 0 w.r.t x, y
0
Q: Proving that the preimage of an open set is open

AjmalWI am trying to learn how to prove that the preimage of an open set is open in general topology. Here is an example that I am not really satisfied with Proposition 3.9 (Book: Essential Topology, Crossley): If $S$ has the discrete topology and $T$ is any topological space, then any function $f:...

0
Q: Question about a ring of polynomials, and the generators of ideals in that ring

The Physics StudentI am studying for an exam, and this is a question from a previous homework that I got wrong. Im not really sure how to start, and any help would be appreciated! Let $R=\mathbb{Z}_7 [x]$ and $I$ be the ideal generated by the polynomial $x^3+2x+1$ a)Let $f$ and $g$ be the elements of $R/I$ repres...

0
Q: Formal proof of Lyapunov stability

CTNTI was trying to solve the question of AeT. on the (local) Lyapunov stability of the origin (non-hyperbolic equilibrium) for the dynamical system $$\dot{x}=-4y+x^2\\\dot{y}=4x+y^2$$ The streamplot below indicates that this actually is true. Performing the change of variables to polar coordinat...

Tagged stability-theory. Formal proof of Lyapunov stability
0
Q: Show equality in distribution

AadLet $X,Y$ be two independent random variables such that $P(X=1)=P(X=-1)=1/2$ and $Y$ is symmetric. Show that $X|Y|=_d Y$.

Short title. Short question. Show equality in distribution
0
Q: Isometry and Lines

MarkIf $f$ is an isometry of the plane and $L$ is a line, prove that $f(L)$ is a line. I know that isometries preserves distance, so that is easy enough. I also know that two distinct point make up a line. Since we know that they share the same distance, I only have to prove that the image is a lin...

Short title. Isometry and Lines
0
Q: Formulae of the Year 2016

JensSoon it's the year 2016. Time to ponder how we can arrange the digits in 2016 to form a valid equation. Use any symbols you like (please explain the less obvious ones). Keep digits in the same order (should this be relaxed?). Examples: $$\lfloor e^2\rfloor + 0 - 1! = 6$$ $$\lfloor\sqrt{\sqrt{20...

0
Q: Can this question be solved by Baye's theorem?

Vikas KumarQ. Of all the graduate students in a university, 70% are women and 30% are men. Suppose that 20% and 25% of the female and male population respectively smoke cigarettes. What is the probability that a randomly selected graduate student is: (a) A woman who smokes. (b) A man who smokes. In my boo...

Welcome to Math.SE, Vikas Kumar. Words such as question are uninformative in titles. Please edit the title so that it better describes the specifics of your question. Do not hesitate to make it longer or include a formula if needed. More tips here. (autocomment)Normal Human 20 secs ago
0
Q: Possible error from Royden's Real Analysis (4th ed.)

AJYI'm using Royden's Real Analysis for a class (4th edition), and there's one line that I'm not sure if it should say what it does. At the top of pg. 110, when they're proving the Vitali covering lemma, on the line labeled (4), it says that $E \setminus \bigcup_{k = 1}^{\infty} I_{k} \subseteq \big...

0
Q: Brief moment from theorem 8.5 on PMA Rudin

Raheem Najib He want to prove that $A$ is open. He take any $x_0\in A$ and showed that exists $\varepsilon:=R-|x_0|$ such that for any $x\in N_{\varepsilon}(x_0)$ we have $f(x)=0$. $A$ is the set of all limit point of $E$ in $S$. How he conclude that $N_{\varepsilon}(x_0)\subset A$? Can anyone explain thi...

0
Q: Get a solution of of 2nd order PDE.

SaimaI have: (x-y) uxy - ux + uy = 0 ----(1) i have to get the solution: u(x,y) = (x-y)^-1 [X(x) - Y(y)] to solve this at first let, v = (x-y)u u = v/(x-y) then i can derive the derivatives of u: ux = -...

Question contains please. Get a solution of of 2nd order PDE.
0
Q: estimator and variance

user298960Let $\mu,\alpha_n:\mathbb R^+\to \mathbb R$ continuous function with $\mu$ bounded function. Let $N^{(n)}$ the trajectory of a Poisson process with intensity $(\alpha_n \mu)(t)$. Let $0=T_0^{(n)}<T_1^{(n)}<..$ jumps of $N^{(n)}$. Let $M_n(t)=\sum_{i=1}^{N_t^{(n)}} \frac {1} {\alpha_n (T_i^{(n)})...

 
7:39 PM
0
Q: Digital Circuit Design Job Market Question Appropriate

AdamI have a question regarding the future of Digital Circuit Design/VLSI as layout becomes more automated. I would like insight from professionals in industry, would this be an appropriate question to ask of EE stack exchange? If not, where can I ask such a question?

 
0
Q: Different proof of fundamental theorem of calculus

luka5zI found this proof sketch in Rosenlicht's book. I get the overall idea, but i don't get why we can write $f(z)$ in the way shown in c). I would be grateful if someone explained this proof to me. Thank you.

0
Q: Jordan decomposition thm follows from Riesz-Kakutani Representation thm, on X compact and Hausdorff?

Hong-Bin ChenThis problem 49 from chapter 9 of Royde & Fitzpatrick's Real Analysis 4th edition. Let X be a compact Hausdorff space. show that the Jordan decomposition Theorem for signed Borel measures on B(X) [the Borel sigma-algebra] follows from the Riesz[-Kakutani] Representation Theorem for the dual of ...

This site uses MathJax formatting of formulas. More tips here. (autocomment)Normal Human 21 secs ago
0
Q: Number of values in square root in different cases

Sumit PalI have two equations: x = √16 x^2 = 16 In first case I think there will be two value of x = +-4. Because -4 * -4 = +4 * +4 = 16 In the second case I am confused. It can also have two values i.e +- 4. But some where I read that in one of the equations return only one value and it will be +4...

Welcome to Math.SE, Sumit Pal. This site uses MathJax formatting of formulas. More tips here. (from a bot)Normal Human 21 secs ago
0
Q: A query about First-Order-Logic Dynamic Logic

Chen LuLet p be an atomic proposition, let a be an atomic program, and let $π = (K, M)$ be a Kripke frame with $K = \{u, v, w\}$ $Mπ(p) = \{u, v\}$ $Mπ(a) = \{(u, v), (u, w), (v, w), (w, v)\}.$ The answer is: In this structure, $u⊨$$¬p∧$<$a$>$p$, but $v⊨[a] ¬p$ and $w⊨[a] p$. Moreover, every stat...

Words such as query do not add information to titles. Please edit the title so that it better describes the specifics of your question. Do not hesitate to make it longer or include a formula if needed. More tips here. (autocomment)Normal Human 21 secs ago
0
Q: Confused about the number of permutations of the Enigma Machine

NikoI recently learned about the Enigma Machine in my cryptography class, but I am a bit confused as to the number of permutations of the wheel settings. According to every article I've read on the matter, the number of different ways the wheels could be set up equals the number of permutations that ...

 
7:55 PM
0
Q: Questions page seems broken when pressed back button on browser

Midhun MPI was looking ios questions recently, and clicked on my profile picture to navigate to my profile page. When I clicked on the back button of my browser the questions page showed something like this: All the right side contents were gone, all hot posts, ads and everything were missing. I tried ...

 
0
Q: Evaluate $\int_L{\frac{z^2}{x^2+y^2}}$

Kristians KuhtaHow can I evaluate $$\int_L{\frac{z^2}{x^2+y^2}}$$, where: L: $$x=\cos{t};$$ $$y=a*\sin{t};$$ $$z=a*t$$ $$0\leq t \leq 2\pi$$ So far I tried it this way: $$I=\int_L{\frac{z^2}{x^2+y^2}}ds=\int_0^{2\pi}{(\frac{a^2t^2}{\cos^2{t}+a^2*\sin^2{t}}\sqrt{(\frac{d(\cos{t})}{dt})^2 + (\frac{d(a*sin{t}...

 
8
Q: Careers Unificintegration: Jobs on Stack Overflow

Jason Punyon TL;DR: In a few days we're gonna start rolling out Jobs on Stack Overflow. If you'd like to help test it early click here. Report bugs here on Meta Stack Overflow with the jobs tag. <3 In 2009 Stack Overflow embarked on a mission to get programmers jobs they love. We star...

0
Q: What happened to my new tabs?

dbuggerStarted using the new nav a few weeks ago. As a good do-bee I customized my tabs on the three computers I use to access SO. Everything was peachy until today. Clicked on a tab, looked at the questions, then returned to the main site by clicking on the logo. My tabs have been replaced by the def...

 
0
Q: Tangent cone at a point.

Lina45Let $ U \subset X $ be an open neighbourhoud of $x$, let $i : U \to \mathbb{A}^n$ be a closed immersion and let $U$ be defined by the ideal $I \subset k[ X_1 , \dots , X_n ]$. There is no loss of generality un supposing $i(x) = (0, \dots , 0) \in \mathbb{A}^n$. Given $f \in k[X_1 , \dots , X_n ]$...

0
Q: The infinite sum of integral of positive function is bounded so function tends to 0

ManikSinLet $f_n(x)$ be positive measurable functions such that $$\sum_{n=1}^\infty \int f_n \lt \infty$$ show that $f_n \to 0$ almost everywhere Attempt: let $K = \sum_{n=1}^\infty \int f_n$ and $S_m = \sum_{n=1}^m \int f_n$ so there is $\forall \epsilon \gt 0 $ a $L$ such that $\forall m \gt L $ we ...

 
8:17 PM
0
Q: Heat Equation with Neumann Boundary Conditions

user286417I was wondering if I was on the right track with this problem given that I have not had much guidance with them. $u_{t}=u_{xx}+2u$ $u_{x}(0,t)=u_{x}(1,t)=0$ $u(x,0)=cos(\pi*x)+cos(3*\pi*x)$ So far I have: $u(x,t)=X(x)w(t)$ $X(x)w'(t)=X''(x)w(t)+2X(x)w(t)$ $\frac{w'(t)}{w(t)}=\frac{X''(x)}{...

Tagged pde, differential-equations. Heat Equation with Neumann Boundary Conditions
 
8:28 PM
0
Q: Generalised version of le Cam's Third Lemma

STFI'm confused with the generalised version of Le Cam's Third lemma presented in Theorem 6.6 of van der Vaart asynptotics Statistics here: https://books.google.co.uk/books?id=Ocg2AAAAQBAJ&pg=PT188&lpg=PT188&dq=van+der+vaart+theorem+6.6&source=bl&ots=Rm4Rw_KJ8H&sig=iXbKRK_HhSX1qQhKjhlGOcLC7Nc&hl=it&...

0
Q: Find transitive relations

AlexDoes this relation are transitive? I know that the relation is reflexive and symmetric but I can not find transitive.

Short title. Short question. Find transitive relations
0
Q: Prove: $\exists !$ $t \in \mathbb{N}$ s.t. $\forall s \in \mathbb{N}$, $(t-9)s = 0$

JabernetI have a basic uniqueness proof to help me work on form: It should be obvious by simple inspection that the statement is true for t=9 and only for t=9. So my proof was this: Let $t=9$ then $9-9=0$, and $0 \cdot s=0$ For the uniqueness, assume that $w,x \in \mathbb{R}$ satisfy the property. Then...

0
Q: An exercise from stein's fourier analysis

user115608I'm trying to solve Exercise 20 of Chapter 5 of Fourier Analysis by Stein. The problem is as follows: Suppose $f$ is of moderate decrease and that its Fourier transform $\hat{f}$ is supported in $I=[-1/2,1/2]$. (a) Prove the following reconstruction formula holds: $f(x)=\sum_{n=-\infty}^{\infty}...

 
2
Q: Searching by zip code loses zip code after postback

JonHWhen searching for a job using the new jobs tab in stackoverflow it seems to lose the location after you select a company. For instance, click the search companies tab. Enter the location 48310. Click on any company such as Quicken Loans..look over Quicken loans then scroll back and click sear...

4
Q: 2015 Community Moderator Election Results

Jon EricsonSoftware Recommendations's first moderator election has come to a close, the votes have been tallied, and the 3 new moderators are: They'll be replacing the existing crew shortly — please thank them for volunteering, and share your assistance and advice with them as they learn the ropes! A...

1
Q: Bounty spelling wrong in tabs?

NicENot sure this is a relevant post for Meta but "bounty" seems to be misspelled in the browser tabs and in the SO tabs.

 
0
Q: solving trigonometric equatuons

melaniesinx+1=cosx, xE[-pi,pi] How do you solve by squaring both sides? the solution is x={-pi/2,0} and I do not understand how by subbing -pi back into both sides of the equations makes them uequal, and the same for positive pi. Which equation are you subbing pi into to check, the original? Thanks

This site uses MathJax formatting of formulas. More tips here. (autocomment)Normal Human 20 secs ago
0
Q: May Proof of Poincaré-Miranda theorem.

MathOverviewI looked for a simple proof Theorema the Poincaré-Miranda based on the intermediate value theorem. Not succeeding, I prepared a proof that was simpler than I thought. The fact that I have not found proof so simple Poincaré-Miranda Theorem in textbooks or articles (but proofs that are based on not...

Tagged proof-verification. May Proof of Poincaré-Miranda theorem.
 
8:46 PM
1
Q: Create PDF in new jobs site does nothing

JonHOn the new jobs site if I go to Create PDF as shown and click on it to generate my PDF it does nothing: Tested both using latest version of chrome and IE. This is found in the edit cv tab of jobs.

 
0
Q: find truth tables and K-Map

JavaCoderthere are four inputs for the funtion f. the output is 1 if and only if two or three of the four inputs are "1". a. create a truth table for the function f b. minimize it using K-Maps c. implement the minimized function using AND and OR gates. It was a review question and i missed the sess...

Short title. Question contains please. find truth tables and K-Map
 
0
Q: Make college / university stay on it's own line in cv

JonHFor the jobs cv in the education section when you add / edit a section you can give the degree which has it's own text box. Along with that you can enter the university / college that you obtained this degree. It looks like this: Looks great, however, when you save your changes the universit...

0
Q: Analytics in Jobs seems broken

Steven VI have a feeling there's some analytics data that is going unrecorded in the new 'Jobs' section. Screenshot from a Windows 10 machine running Chrome 47. On the positive side, these errors don't seem to hampering my browsing of the section!

0
Q: favorite jobs clicked right container still telling me to view all favorites

JonHI've got my eye on a few places from the jobs section by making them a favorite. After you've reached more then 3 favorites the right container adds a link to view all favorites as shown: After clicking the link I am taken to my job favorites, I think the site no longer needs to tell me I hav...

 
0
Q: Quick doubt on a negation of a statement.

MonoliteThe statement I want to negate is: there exists a $\delta > 0$ s.t. $f(x) > 0$ $\forall x \in A$ where $|x-x_0| < \delta$ and $x \ne x_0$ The negation I think is correct is: for all $\delta > 0$ there exist an $x \in A$ s.t. $f(x) \le 0$ where $|x-x_0| < \delta$ and $x \ne x_0$. But a friend of...

Words such as doubt do not add information to titles. Please edit the title so that it better describes the specifics of your question. Do not hesitate to make it longer or include a formula if needed. More tips here. (autocomment)Normal Human 21 secs ago
0
Q: A geometry question

geometerLet $I$ be the incenter of triangle $ABC$. Let $D,E,F$ be the intersections between the incenter and sides $BC,CA,AB$ respectively. Let $M$ be the midpoint of $EF$ and let $Q$ be the second intersection between $AD$ and the incircle. Show that $MIDQ$ is a cyclic quadrilateral. I have tried some...

Welcome to Math.SE, geometer. Words such as question do not add information to titles. Please edit the title so that it better describes the specifics of your question. Do not hesitate to make it longer or include a formula if needed. More tips here. (autocomment)Normal Human 21 secs ago
0
Q: About the solutions of the Bessel differential equation and the modified Bessel differential equation

BowParkThe modified Bessel differential equation can be obtained by replacing $x$ with $ix$ ($i$ is the imaginary unit) in the Bessel differential equation. If the general solution of the latter is $$f(x) = A J_{n}(x) + BN_n(x)$$ While the general solution of the modified Bessel differential equation ...

0
Q: Measurable function understanding

Al jabraA function f on X is said to be $\gamma$ measurable if for every real number $\alpha$ the set $\{x \in \gamma |f(x) > \alpha)$ Then I have that the following statements are equivalent for a function f on X to $\mathbb{R}$. (a) For every $\alpha \in \mathbb{R}$ the set $A_\alpha= \{x \in \gamma...

0
Q: How to prove ((^B -> ^A) -> (^B -> A)) -> B with resolution

swordgitthe ^ symbol represent negation. I have proved so far... (^B -> ^A) -> (^B -> A), ^B |- "bot" <=> ^(B v ^A) v (B v ^A), ^B |- "bot" and then I have no clue for next step til getting the answer please help! thank you!

This site uses MathJax formatting of formulas. More tips here. (autocomment)Normal Human 21 secs ago
0
Q: Independent events (conditions)

nicoleAn insurance company insures E drivers under age 24 and D drivers over 24 years old. Of these drivers, e under 24 and d over 24 had an accident in a 1 year period. A driver insured by this company is chosen at random. Let A be the event that this driver is under 24 and B be the event that this dr...

0
Q: Isometries and Lines

MarkIf ff is an isometry of the plane and LL is a line, prove that f(L)f(L) is a line. I know that isometries preserves distance, so that is easy enough. I also know that two distinct point make up a line. Since we know that they share the same distance, I only have to prove that the image is a line...

Short title. Isometries and Lines
0
Q: Proving the Recursion Theorem (Hungerford)

Andy TamThe proof below was taken from Hungerford. I can see how $R = \cap_{Y \in G} \, Y$ defines the desired $\varphi: \mathbb N \rightarrow S$, but I don't see why he had to go through so many mechanics to do so. I can define $\varphi: \mathbb N \rightarrow S$ as follows: \begin{align} \varphi(0) &...

 
9:14 PM
0
Q: Allow discussion with employers (At least one?) when submitting a job application

JonHWhen I post an application to a potential employer I can see my initial submit via the messages link from the jobs tab. As it stands, the UI blocks any type of communication from the candidate to the employer. This kind of makes sense as you guys probably do not want to harass / bother the empl...

0
Q: When applying for a job js error causing me not to upload a file

JonHThe new jobs section if I click the shiny blue Apply button for jobs that allow stackoverflow to handle submitting cv's is not letting me upload a file. The page is presenting some js errors indicating drop is not found Console window:

 
0
Q: Gauss Quadrature Proof.

antwontheswan2Let $x_0<x_1< ... < x_n$ be the roots of an n+1 degree orthonormal polynomial $\phi_{n+1}$ with respect to the inner product: $$\langle g,h \rangle = \int_a^bw(x)g(x)h(x)dx$$ and $$p_n= \sum_{j = 0}^nf(x_j)L_{n,j} \in \mathbb{P_n}$$ be the lagrange interpolating polynomial for the given data....

0
Q: Find the Area bounded by $y=-x^2-4$ $y=x+2$

CetshwayoI set the two polynomials equal to each other and after multiplying everything by -1 I got x^2+x-2=0 However, the graph of the polynomial is concave up. Why?

 
0
Q: Print link does nothing in jobs site

JonHIf I want to print my cv using the jobs tab I used to be able to click print and get a print preview dialog allowing me to select a printer. As it stands nothing happens when I click the Print hyperlink: This is found in the edit cv section tab of the jobs area:

 
0
Q: Proof minimum of function is given by normal equations

geraldI have to proove the following: Let $A$ be a mtrix of size $m, n$ with rank $n$. Prove that the minimum for $f(x) = ||Ax -b||^2$ is given by the normal equation. I have absolutely no idea - would be great, if someone could help :)

Tag (proof-explanation) should not be the only tag a question has. Please add a tag for a subject area to which the question belongs. (from a bot)Normal Human 21 secs ago
0
Q: Find volume of the region

GuestCalcFind the volume of the region when B is the region bounded by the cylinder $$x^2 + 3z^2 = 9$$ and the planes $$y = 0$$ and $$x + y = 3$$. $$ \int_{-3}^{3} \int_{0}^{3-x}\int_{-sqrt{(9-x^2)/3}}^{sqrt{(9-x^2)/3}} \;dz \;dy\;dx $$ Is my integral correct?

0
Q: Find an x such that Q(x) = Q(2^0.5, 3^0.5, 5^0.5).

Yankees2Find an x such that Q(x) = Q(2^0.5, 3^0.5, 5^0.5). For my abstract algebra class. Don't really know where to start, or how to finish for that matter.

0
Q: Extremum of function on l1

student93How to prove that $f: l_1 \to \mathbb{R}$ s.t. $$f((x_n)_{n \in N})= \sum_{n=1}^{\infty} (\frac{1}{n} x_{n}^{2}+ x_{n}^{3})$$ is a function of class $C^{\infty}$ and $f'(0)=0$ and $f''(0)(h,k)>0$ for $h,k \in l_1 \setminus \{0 \}$ but $f$ has no extremum at $0$ Let $x=((x_n)_{n \in N}) $ and $h=...

Consider replacing (analysis) with a more specific tag for the relevant branch of analysis. (from a bot)Normal Human 21 secs ago
0
Q: Find $\sum\limits_{n=1}^{\infty}\frac{n^2}{2^n}$ using the function $f(x)=\frac{1+x}{(1-x)^3}$

user_99Find $\sum\limits_{n=1}^{\infty}\frac{n^2}{2^n}$ using the function $f(x)=\frac{1+x}{(1-x)^3}$ Power series representation of $f(x)$ is $\sum\limits_{n=1}^{\infty}n^2x^{n-1}$. Question: Why is $\sum\limits_{n=1}^{\infty}\frac{n^2}{2^n}=\frac{1}{2}f\left(\frac{1}{2}\right)$? Doesn't it has to be...

0
Q: Exponential in the category Relation

Meggy LeeIs it possible to define exponential and currying in the category Relation? If not, what is the reason that we cannot?

 
9:46 PM
0
Q: Are broad explaination type questions on topic?

FunkyguyEvery once in a while, I'll stumble upon a high-rep member who will have written a question and then a very lengthy answer that very effectively answers it. Case and point, a great Q/A by Olin I want to make sure, are these on topic given they are usually inherently broad questions or is there ...

0
Q: Create Markdown flickers job site then does nothing

JonHThe jobs section under edit cv is a neat little icon to Create Markdown of your cv. If it is clicked the entire site flickers and takes you to an empty page with just the stackoverflow header. Here is a screen capture of it's location:

0
Q: Inconsistency between search jobs and search companies when selecting filters

JonHIf I'm on the search jobs tab and I hover over location and check off the checkboxes Allows Remote, Offers Relocation, and Offers Visa Sponsorship and Search I get something that looks like this: If I do the same exact thing except on the search companies tab we get a different output: The ...

0
Q: Are questions about good coding style on topic?

Kyle StrandI flagged this SO question as off-topic with the comment "I think this would be a better fit for Programmers.SE, but that's not one of the migration options." This flag was declined with the message "a moderator reviewed your flag, but found no evidence to support it." I'm fairly certain that th...

0
Q: New Jobs page shows "Create a Developer CV" on SO profile when I already have one

AndyI've opt'd into the new Jobs feature. However, I how have a giant "Create a developer CV" on my SO profile. I already have a CV. The button it self, and the "Edit CV" 'tab' both take me to my CV. This button should be either removed or renamed if the CV already exists.

2
Q: Are questions about the Google Analytics reporting interface on topic

Eike PierstorffI work pretty much exclusively on the Google Analytics tag. I'm not much a stickler for the rules, but when people ask about the reporting interface I usually leave a comment that this is off-topic (since questions about the interface are not about programming, software algorithms, tools specifi...

 
0
Q: solving trigonometric equations part 2

melanieThis is the second part of my last question sinx+1=cosx, XE[-pi,pi] The question asks to solve using a half angle identity and in the solution it states to subtract 2pi from 3pi/2 resulting in one of the solutions being -pi/2 along with another solution of zero. I am confused how you would know...

This site uses MathJax formatting of formulas. More tips here. (from a bot)Normal Human 21 secs ago
0
Q: If f: X→R, where X is some metric space, be continuous. Prove that if f(a)=0 for every b ∈ the closure of A.

SherlockEddyIf f: X→R, where X is some metric space, be continuous. Prove that if f(a)=0 for every element a ∈ A ⊂ X, then f(b)=0 for every b ∈ the closure of A. Thanks.

0
Q: Expectation of a function of Brownian motions

DanielI would like to how I can compute this expectation and get the answer that is given. All terms W indicate a Wiener process. E_t[W_s^3]=E_t[(W_t+(W_s-W_t))^3]=W_t^3+3W_t(s-t)

Welcome to Math.SE, Daniel. This site uses MathJax formatting of formulas. More tips here. (from a bot)Normal Human 21 secs ago
0
Q: No simple group of order 2016

jimvb13The year 2016 is coming up, so it makes me wonder: how do you prove that there is no simple group of order 2016? (without invoking the 10,000 page theorem on the classification of simple groups)

 
10:05 PM
1
Q: How can I stalk a user?

FabbyEarlier today, I ran across this user who had a few (~5) answers and after a few constructive comments directed to it and removing the link bait from or voting to delete its answers, I suddenly noticed there were only 3 answers left... One I can still find as I still had it open in a tab so th...

 
0
Q: plot an array in Maple

jimI have the following data list in a file, say mydata.txt 0 -2.900125720 -253.200 1 -5.512974510 -253.800 9 -398.4569435 -253.16 10 -748.4988836 -253.19 I read the file in as readdata(mydata.txt, float, 3) and this generates a data list, say mydatalist := [[0., -2.900125720,...

Consider adding a tag for a broader subject area to which the question belongs. Some of these tags might fit. (from a bot)Normal Human 21 secs ago
 
10:17 PM
0
Q: Interpretation of question 1 in Appendix of Hatcher's Algebraic topology

akgjhI am trying to interpret the first question statement of the Appendix in Hatcher's topology A covering space of a $CW$ complex is also a $CW$ complex, with cells projecting homeomorphically onto cells. Consider a $CW$ complex with, for example, $2$ $k$-cells in each dimension $k$, for $k \le ...

0
Q: How to find a convex decomposition for two positive semi-definite matrices

silhouseenter image description here Any help and comments will be greatly appreciated.

0
Q: Analytic Functions with certain conditions

herashefatFind all functions f(z) which are analytic in the region |z|<1 (or equal to 1) and are such that f(0)=3 and |f(z)|<3 (or equal to 3) for all z such that |z|<1...... How???

0
Q: Prove confex functions

lars111I have to prove the following things: a) Assume that f is a convex function on S. Prove that the set ${x element S; f(x) <a$ is a convex set. b) Prove that the sum of two convex functions is a convex function c) Prove that an affine function is convex Proving things is completely new for me :)

Short title. Tagged proof-explanation. Prove confex functions
0
Q: Prove constrained problem has no duality gap

antifxFor an exam I have to prove that if a constrained problem has no duality gap for some $(l,g)$ and $x$, then $x$ is a global minimum point for the constrained problem --> Do you think, an example is sufficient?

Questions tend to get more attention when they have a tag for a broad area of mathematics relevant to the question. Some of these tags might fit. (from a bot)Normal Human 21 secs ago
 
10:51 PM
0
Q: Expected outcome for one investment with two distinct outcomes and known probability

BigkidA single investment has two distinct outcomes with known probabilities. For example, this investment either returns 5% (with a .35 probability or returns 12% (with a .65 probability).

0
Q: Prime Concatenation Order

Alex A.Consider the following procedure. Given an integer $n \geq 2$, obtain the canonical prime factorization of $n$, i.e. $\prod_{i=1}^k p_i^{e_i}$. Take the distinct factors $p_i$ and list them in ascending order. Concatenate them into a new integer. That is, 2 and 5 becomes 25, 3 and 7 becomes 37, ...

0
Q: Fourier Transform PDE 2D

AnonymousJIs it possible to take the Fourier transform of "cross-sectional plane"; in other words, is $F\{\frac{\partial^{2}{u}}{\partial{x}\partial{y}}\}$ possible where $F\{\cdot\}$ is the Fourier transform? I know how to solve $\left[F\{\frac{\partial^{2}{u}}{\partial{x}^{2}}\} + F\{\frac{\partial^{2}{u...

-1
Q: Help!!! Expected Value Problem!!

aperetzf(x,y)=4xy x,y[0,1] Find E(x-y) and v(x).

Welcome to Math.SE, aperetz. Words such as help are uninformative in titles. Please edit the title so that it better describes the specifics of your question. Do not hesitate to make it longer or include a formula if needed. More tips here. (from a bot)Normal Human 21 secs ago
0
Q: Integrability of $\frac{1}{|x|^d}$

CYCFor $x \in \mathfrak R^d$,why is $\int_\limits{\{x; |x|\geq 1\}} \frac{1}{|x|^d} dx = \infty$ in Lebesgue integral? It's hinted to apply Tonelli Theorem (Fubini Theorem) and use the fact that $\frac 1 x$ is not integrable over $[1,\infty)$, but I don't know how.

0
Q: Graph with a range of (-∞, 0]

ChakeI'm trying to figure out a graph with a range of (-∞, 0]. I can't seem to figure it out. Keep in mind that I tried telling him right away -(x)^2, but he said that he wanted one that wasn't a [0, ∞) (E.G. absolute value, even exponentials, even roots, etc.) just negated. Any ideas?

Short title. [Graph with a range of (-∞, 0]](math.stackexchange.com/q/1577574)
0
Q: Banach Space Closed Subspace

Anthony PeterLet $ \mathcal B$ be a Banach Space. Fix $z \in \mathcal B$. Consider the set $$A :=\{y-z : y \notin \operatorname{span} \{z\}, y \in \mathcal B\}.$$ Is it true that $\alpha z \notin \overline{A}$ for any $\alpha \in \mathbb{C}$? I'm looking for a counterexample or a hint about how to prove i...

Short title. Tagged proof-writing. Banach Space Closed Subspace
0
Q: Describe the open sets of the Cartesian product of metric spaces.

karhasI want help with the proof of this theorem Consider the metric spaces $(\mathbb{X}_j,d_j), j=1,\dotsc,n$. Let $\mathbb{X}=\mathbb{X_1}\times\cdots\times\mathbb{X_n}$ The map $E:\mathbb{X}\times\mathbb{X} \to \mathbb{R}$ with $$E(x,y)= \left[ \sum_{j=1}^n\big( d_j(x_j,y_j ) \big)^2 \...

0
Q: Floating Point and machine error

Jasimud Let $a$ and $b$ be to real arbitrary real numbers, show that the relative error that you made by computing $a^2b$ with floating point arithmethic is bound to $5\epsilon + O(\epsilon^2)$, with $\epsilon$ the machine error. As $a$ and $b$ are real numbers I can write them as: $a = a(1 + \d...

Tagged proof-verification. Floating Point and machine error
 
11:13 PM
0
Q: Low quality/ off topic questions being up voted all the time

Peter MMany times I see a reasonably new user at SO ask a really bad or off topic question (for example this one that prompted me to ask this question on meta). That's understandable, but what I don't get is that pretty well straight away these types of questions get up-voted. Is this a feature of SO...

 
11:27 PM
0
Q: Legendre symbol of $-n$

LudwwwigLet $x, y, n, p$ be positive integers such that $x^2 + ny^2 = p$, where $p$ is a prime such that $p \neq n$. Show that the Legendre symbol $(-n/p) = 1$.

Short title. Short question. Legendre symbol of $-n$
0
Q: Question about isomorphism of Galois groups

Werner Germán BuschLet $E/F$ and $E/F'$ and $E/F\cap F'$ be separable and normal field extensions. I am trying to show that $Gal(E/F\cap F') \simeq Gal(E/F).Gal(E/F')$

Words such as question do not add information to titles. Please edit the title so that it better describes the specifics of your question. Do not hesitate to make it longer or include a formula if needed. More tips here. (from a bot)Normal Human 20 secs ago
0
Q: Let $T$ be such that for every vertices $u,v$ there exists a unique walk joining them, then $T$ has no cycles and $T+e$ has exactly 1 cycle.

YoTengoUnLCDI'm trying to understand a part of the proof of the following Theorem Let $T$ be such that for every vertices $u,v$ there exists a unique walk joining them, then $T$ has no cycles and $T+e$ has exactly 1 cycle. Proof. If $T$ had a cycle, say $<v_1,e_1,v_2,e_2,...,v_1>$ then there wo...

 
0
Q: Does a sandbox page exist?

HohmanfanI am eager to try some some types of formatting, like in-line code, image customization, HTML support and a lot of other things to improve my posts. Does Stack Exchange have a sandbox page where I can try this without changing existing answers or questions?

 
11:50 PM
0
Q: Should I repeat other answers to form a complete one?

dan08If a question has multiple solutions and someone has already provided one solution. Should I include that solution in my answer in an attempt to create one complete answer? Or is it better to just present the other possible solution?

 
0
Q: Irrational expected value with rational definitions

user1657355Is there some probability distribution that can be implemented/defined/etc. without irrational numbers such that it returns 1 an irrational proportion P of the time and 0 the rest of the time, for any irrational probability P? If not, for what irrational P can this be done? I am specifically tryi...

Welcome to Math.SE, user1657355. This site uses MathJax formatting of formulas. More tips here. (from a bot)Normal Human 20 secs ago
 
00:00 - 16:0016:00 - 00:00

« first day (43 days earlier)      last day (531 days later) »